Difference between revisions of "2006 AMC 12A Problems/Problem 4"

m
(Duplicate, AMC 10 box, LaTeX spacing)
Line 1: Line 1:
 +
{{duplicate|[[2006 AMC 12A Problems|2006 AMC 12A #4]] and [[2006 AMC 10A Problems/Problem 4|2008 AMC 10A #4]]}}
 
== Problem ==
 
== Problem ==
 
 
A digital watch displays hours and minutes with AM and PM. What is the largest possible sum of the digits in the display?
 
A digital watch displays hours and minutes with AM and PM. What is the largest possible sum of the digits in the display?
  
<math>\mathrm{(A)}\ 17\qquad\mathrm{(B)}\ 19\qquad\mathrm{(C)}\ 21\qquad\mathrm{(D)}\ 22\mathrm{(E)}\  23</math>
+
<math>\mathrm{(A)}\ 17\qquad\mathrm{(B)}\ 19\qquad\mathrm{(C)}\ 21\qquad\mathrm{(D)}\ 22\qquad\mathrm{(E)}\  23</math>
  
 
== Solution ==
 
== Solution ==
Line 10: Line 10:
 
== See also ==
 
== See also ==
 
{{AMC12 box|year=2006|ab=A|num-b=3|num-a=5}}
 
{{AMC12 box|year=2006|ab=A|num-b=3|num-a=5}}
 +
{{AMC10 box|year=2006|ab=A|num-b=3|num-a=5}}
  
 
[[Category:Introductory Number Theory Problems]]
 
[[Category:Introductory Number Theory Problems]]

Revision as of 00:04, 28 April 2008

The following problem is from both the 2006 AMC 12A #4 and 2008 AMC 10A #4, so both problems redirect to this page.

Problem

A digital watch displays hours and minutes with AM and PM. What is the largest possible sum of the digits in the display?

$\mathrm{(A)}\ 17\qquad\mathrm{(B)}\ 19\qquad\mathrm{(C)}\ 21\qquad\mathrm{(D)}\ 22\qquad\mathrm{(E)}\  23$

Solution

From the greedy algorithm, we have $9$ in the hours section and $59$ in the minutes section. $9+5+9=23\Rightarrow\mathrm{(E)}$

See also

2006 AMC 12A (ProblemsAnswer KeyResources)
Preceded by
Problem 3
Followed by
Problem 5
1 2 3 4 5 6 7 8 9 10 11 12 13 14 15 16 17 18 19 20 21 22 23 24 25
All AMC 12 Problems and Solutions
2006 AMC 10A (ProblemsAnswer KeyResources)
Preceded by
Problem 3
Followed by
Problem 5
1 2 3 4 5 6 7 8 9 10 11 12 13 14 15 16 17 18 19 20 21 22 23 24 25
All AMC 10 Problems and Solutions